rbetita
Thanks Received: 0
Forum Guests
 
Posts: 5
Joined: May 10th, 2011
 
 
 

Q4 - Chapin: Commentators have noted

by rbetita Fri Sep 16, 2011 7:03 pm

Hi,

Just wanted to know if I was on track with the reasoning for #4.

I have what I thought was the argument core:

P-Won pluralities not majorities
P-won when moderate parties were arguing amongst selves

© (Electoral successes) by extremist parties pose no threat to democracy

I originally selected E, but when I read it again I realized that this addressed "People" and didn't really help us bring us to the conclusion why democracy wasn't threatened.

I think A is the correct answer because it addresses the fact that since the extremists won a pluralities, this doesn’t affect political arrangements and so democracy would not be affected

Are the other choices incorrect because:

(E) talks about people supporting _ we want to know why democracy is not threatened

(B) who cares about multiparty systems

(c) we don’t care about strength of government, but would the answer be more correct if it said: "countries in which extremist parties win pluralities have never had a history of the previous democratic regime changing to an extremist government"

(D) again, arguing doesn't allow us to get to our conclusion

THanks!
User avatar
 
ohthatpatrick
Thanks Received: 3808
Atticus Finch
Atticus Finch
 
Posts: 4661
Joined: April 01st, 2011
 
This post thanked 5 times.
 
 

Re: Q4 - Chapin: Commentators have noted

by ohthatpatrick Mon Sep 19, 2011 1:42 pm

Awesome dissection!

You were on point with everything. Keep in mind that on Sufficient Assumption, you have a very focused goal: "Prove the Conclusion"

Since our conclusion here was that "these extremist victories pose NO threat to democracy in those countries", and the premises never defined for us what would or wouldn't pose a threat to democracy, we need our answer to include the idea that "no threat to democracy is posed".

As you said, the two things we knew about the extremist victories were:
- pluralities, not majorities
- only won when moderates were fighting with each other

So there are only two POSSIBLE ways to prove this conclusion:

1. If extremists won pluralities, not majorities --> pose no threat to democracy in their country

2. If extremists only won when moderates were fighting with each other --> pose no threat to democracy in their country

The common denominator to those two options is the "pose no threat to democracy".

(A) is tricky in that it doesn't give us "pose no threat to democracy" verbatim. Instead, it "overfills the arrow", going beyond what we need and saying it poses no threat to any political arrangement.

The way you fixed (C) was genius, although technically we'd have to say "have never and will never".

Note that 99% of the correct answers on Sufficient Assumption are of conditional, universal, definitive strength.

So "sometimes" in (C), "sometimes" in (D), and "not always" in (E) are huge red flags that make them very unlikely choices.

Hope this helps. Good work!
 
u2manish
Thanks Received: 0
Forum Guests
 
Posts: 24
Joined: November 03rd, 2011
 
 
 

Re: Q4 - Commentators have noted with concern recent electoral s

by u2manish Tue Dec 20, 2011 6:45 am

Which one would qualify as a necessary assumption.(D)..would it? Could some take a shot at this please..! just trying to understand the distinction..
Thanks,
M
User avatar
 
ManhattanPrepLSAT1
Thanks Received: 1909
Atticus Finch
Atticus Finch
 
Posts: 2851
Joined: October 07th, 2009
 
This post thanked 3 times.
 
 

Re: Q4 - Commentators have noted with concern recent electoral s

by ManhattanPrepLSAT1 Fri Dec 23, 2011 7:21 pm

The distinction between a sufficient assumption and a necessary one is that if you add a necessary assumption to the argument you may not guarantee the conclusion will follow, whereas if you add a sufficient assumption, you will.

Necessary Assumptions are required for the conclusion to follow, so without the assumption the argument's reasoning will not follow.

Generally, Necessary Assumptions are more restrained in that they represent weaker, vaguer, or more limited statements (though not always). Sufficient Assumptions are frequently strong and bold statements that may be stronger than you need to guarantee the conclusion, but will definitely ensure the conclusion's validity.

In this case the language of (C), (D), and (E) that Patrick pointed to above is more typical of a Necessary Assumption. While answer choice (E) does not address the strength of a democracy, both answer choices (C) and (D) seem to be pretty close. But I don't think either are actually necessary the argument's reasoning, since neither address a gap between the evidence and the conclusion.

So I'd suggest that answer choice (D) is close, but not quite.
 
griffin.811
Thanks Received: 43
Atticus Finch
Atticus Finch
 
Posts: 127
Joined: September 09th, 2012
 
 
 

Re: Q4 - Chapin: Commentators have noted

by griffin.811 Fri Jan 11, 2013 9:19 am

Just adding my 2 cents here. this was already well discussed it seems, but for my own sake, here's an explanation of why (D) is incorrect.

1. (D) as someone has said is not necessary to substantiate our core. We are told that "furthermore extremists have only won when mods were fighting each other" but does this mean they can ONLY win if mods are fighting each other? absolutely not! just because something was that way in the past, doesnt mean it must also be that way in the future. I like to think of it like those investing commercials..."Past fund performance is not indicative of future fund performance..."

2. We all should have a really really big issue with the use of the words "pose NO threat" being used in the stim, and (D) saying "SOMETIMES mods will put aside differences..." Really, this is the best you could do LSAT writers? Lets consider the fact that if Mods only put aside differences SOMETIMES, then there must be at least sometimes they dont. In these cases, couldnt extremists pose a threat? Of course, so this answer is incorrect.

I actually got this incorrect myself, but won't happen again.

MLSAT:1
LSAT Writers:0 :twisted:
 
hychu3
Thanks Received: 3
Vinny Gambini
Vinny Gambini
 
Posts: 20
Joined: June 01st, 2013
 
 
 

Re: Q4 - Chapin: Commentators have noted

by hychu3 Sun Nov 17, 2013 1:14 pm

Hello,

I'd like to fix why (B) is incorrect for future reference.

A previous poster (who's probably attending some law school by now) wrote that (B) is incorrect because "multiparty political system" is irrelevant.

Actually, it is relevant. The stimulus says that extremist parties achieve electoral success only when the moderate parties were preoccupied with arguing among themselves. So, those countries where extremists parties found some success are those countries with multiparty political systems (at least two moderate parties + extremist party + possibly other parties as well).

The real flaw with (B) is that it is making a wrong comparison. It's early in the test, and the test makers set a trap for those of us who are rushing. If there were only two moderate political parties (for example, the US has only 2 parties with any seats in the Congress) before the extremist party in question took some seats in congress, then (B) indeed guarantees that democracy is not threatened. We're switching from two-party to multiparty, and (B) says that the latter is more democratic.

But the key is that there could be three or more (moderate) parties already when the extremist party took some seats. In that case, (B) offers nothing.

Note also that the correct answer, (A), does not use the key word "democratic."
User avatar
 
ManhattanPrepLSAT1
Thanks Received: 1909
Atticus Finch
Atticus Finch
 
Posts: 2851
Joined: October 07th, 2009
 
 
 

Re: Q4 - Chapin: Commentators have noted

by ManhattanPrepLSAT1 Wed Nov 20, 2013 9:18 pm

I like your attention to detail hychu3! I think you've got a very good point here.

After reviewing the question, I think I'd add one more thing. In answer choice (B), it describes a comparison between how democratic multiparty systems and two-party systems are. Notice that this is a statement about what is, and not what might become.

For example, in the past there have been cases when extremist parties came to power and destroyed the existing political system. In that case even though the extremist party came from a multiparty system, it still did pose a threat to democracy.